Đến nội dung

Atu nội dung

Có 52 mục bởi Atu (Tìm giới hạn từ 21-05-2020)



Sắp theo                Sắp xếp  

#439800 $2\sqrt{2}cos2x + sin2x. cos(x+\frac{3\pi...

Đã gửi bởi Atu on 01-08-2013 - 22:17 trong Phương trình, Hệ phương trình Lượng giác

$2\sqrt{2}cos2x + sin2x. cos(x+\frac{3\pi }{4})-4sin(x+\frac{\pi }{4})=0$

Ta có:

$2\sqrt{2}cos2x + sin2x. cos(x+\frac{3\pi }{4})-4sin(x+\frac{\pi }{4})=0$

$\Leftrightarrow 2\sqrt{2}cos2x+sin2x.cos(x+\pi-\frac{\pi }{4} )-2\sqrt{2}(sinx+cosx)=0$

$\Leftrightarrow 2\sqrt{2}cos2x-sin2x.cos(x-\frac{\pi }{4})2\sqrt{2}(sinx+cosx)=0$

$\Leftrightarrow 2\sqrt{2}(sinx+cosx)(cosx-sinx)-\sqrt{2}sinx.cosx.(sinx+cosx)-2\sqrt{2}(sinx+cosx)=0$

$\Leftrightarrow (sinx+cosx)(2cosx-2sinx-sinx.cosx-2)=0$

Đặt $ sinx-cosx=t\Leftrightarrow -sinxcosx=\frac{t^{2}-1}{2}$ ....




#410558 Một bài toán vui , mọi người coi nhé!

Đã gửi bởi Atu on 05-04-2013 - 18:22 trong Các dạng toán khác

Bạn nào giải thích cho mình

Cái này đánh lừa thị giác bạn thôi, gọi cạnh huyền tam giác đỏ là AB, ta giác xanh dương là BC, hình ở trên A,B,C thẳng hàng, ở dưới A,B,C không thẳng hàng ( đơn giản nhất là dựa theo tỉ lệ trên bảng rồi dùng công thức lượng giác chứng minh nó khôn thẳng hàng)




#410016 CMR: $\frac{b}{a+2b}+\frac{c}...

Đã gửi bởi Atu on 02-04-2013 - 22:18 trong Bất đẳng thức và cực trị

Cho a,b,c >0 . CMR:

$\frac{b}{a+2b}+\frac{c}{b+2c}+\frac{a}{c+2a}\leq 1$

$AM-GM$ ngược dấu.

Ta có: $\sum \frac{2b}{a+2b}=3-\sum \frac{a}{a+2b}$

Bài toán trở thành:

$\sum \frac{a}{a+2b}\geq 1$ 

Bđt trên đúng vì áp dụng $C-S$:

$\sum \frac{a}{a+2b}=\sum \frac{a^{2}}{a^{2}+2ab}\geq \frac{(a+b+c)^{2}}{\sum a^{2}+2\sum ab}=1$




#409203 Cho 3 số thực x,y,z

Đã gửi bởi Atu on 30-03-2013 - 21:05 trong Bất đẳng thức và cực trị

Cho 3 số thực x,y,z thoả mãn x+y+z=0 và x+2>0;y+2>0;z+8>0

Tìm  MAX của $A=\frac{x}{x+2}+\frac{y}{y+2}+\frac{z}{z+8}$

Đặt $Max$ $A=k$ , ta cần Cm:

$\frac{x}{x+2}+\frac{y}{y+2}+\frac{z}{z+8}\leq k$

$\Leftrightarrow 1-\frac{2}{x+2}+1-\frac{2}{y+2}+1-\frac{8}{z+8}\leq k$

$\Leftrightarrow 3-k\leq \frac{2}{x+2}+\frac{2}{y+2}+\frac{8}{z+8}$

Áp dụng hệ quả $B-C-S$ ta có:

$\frac{2}{x+2}+\frac{2}{y+2}+\frac{8}{z+8}=\frac{4}{2x+4}+\frac{4}{2y+4}+\frac{4}{z+8}+\frac{4}{z+8}\geq \frac{4^{3}}{2\sum x+24}\geq \frac{8}{3}$

Vậy k cần tìm là $3-\frac{8}{3}=\frac{1}{3}$




#408968 chứng minh bất đẳng thức tam giác

Đã gửi bởi Atu on 29-03-2013 - 21:25 trong Bất đẳng thức và cực trị

2, $\frac{1}{b+c-a}+\frac{1}{c+a-b}+\frac{1}{a+b-c}\geq \frac{2}{a}+\frac{2}{b}+\frac{2}{c}$

Đúng rồi, bài 2 đề sai, chắc đề là vậy:

CM:$\sum \frac{1}{b+c-a}\geq \frac{1}{a}$

  Giải:

Ta có:

$\sum \frac{1}{b+c-a}+\sum \frac{1}{c+a-b}\geq \sum \frac{4}{2c}=\sum \frac{2}{c}$

Suy ra $dpcm$




#408906 $\sum \frac{2}{x^3}+\sum \frac...

Đã gửi bởi Atu on 29-03-2013 - 19:23 trong Bất đẳng thức và cực trị

Cho 3 số thực $x,y,z$ thỏa $x+y+z\leq 3$. Tìm GTNN $\sum \frac{2}{x^3}+\sum \frac{1}{x^2-xy+y^2}$

http://diendantoanho...-dfrac1x2-xyy2/




#408732 Tìm GTNN P=$\left ( x^{2}+\frac{1}{y^...

Đã gửi bởi Atu on 28-03-2013 - 20:51 trong Bất đẳng thức và cực trị

Cho các số dương x,y thay đổi thõa mãn điều kiện: x+y=1. Tìm GTNN của biểu thức:

                 P=$\left ( x^{2}+\frac{1}{y^{2}} \right )\left ( y^{2}+\frac{1}{x^{2}} \right )$

Ta có:

$P=(x^{2}+\frac{1}{y^{2}})(y^{2}+\frac{1}{x^{2}})=2+x^{2}y^{2}+\frac{1}{x^{2}y^{2}}=2+(xy)^{2}+\frac{1}{256(xy)^{2}}+\frac{255}{256(xy)^{2}}$

Áp dụng $AM-GM$:

$(xy)^{2}+\frac{1}{256(xy)^{2}}\geq \frac{1}{8}$

$\frac{255}{256(xy)^{2}}\geq \frac{255}{(4(x+y)^{2})^{2}}=\frac{255}{16}$

Cộng lại, suy ra $min P=\frac{289}{16}$

 


Mình vẫn chưa hiểu cái BĐT thứ 2 bạn ơi. giải thích mình vs

 

Ta có: $4xy\leq (x+y)^{2}\Rightarrow (4\times 4xy)^{2}\leq (4\times (x+y)^{2})^{2}=16$

$\Rightarrow \frac{255}{(16xy)^{2}}\geq \frac{255}{16}$




#408466 $\sum (a+1-\frac{1}{b})(b+1-\frac...

Đã gửi bởi Atu on 27-03-2013 - 20:52 trong Bất đẳng thức và cực trị

Chứng minh bất đẳng thức sau với mọi a,b,c >0 : $\sum (a+1-\frac{1}{b})(b+1-\frac{1}{c})\geq3$

Đặt $a+\frac{1}{b}-1=x;\, b+\frac{1}{c}-1=y;\: c+\frac{1}{a}-1=z$

Theo nguyên lí Đi-rích-lê thì ít nhất 2 trong 3 biểu thức $x-1$; $y-1$; $z-1$ cùng dấu.

Giả sử 2 biểu thức đó là $x-1$ và $y-1$

Ta có:

$(x-1)(y-1)\geq 0\Rightarrow xy\geq x+y-1$ (1)

Lại có:

$x+y=a+\frac{1}{b}-1+b+\frac{1}{c}-1\geq a+\frac{1}{c}=\frac{(a+\frac{1}{c})(c+\frac{1}{a})}{c+\frac{1}{a}}\geq \frac{4}{c+\frac{1}{a}}= \frac{4}{z+1}$

$\Rightarrow (x+y)(z+1)\geq 4\Rightarrow (x+y-1)+z(x+y)\geq 3$ (2)

(1) và (2) suy ra $\sum xy\geq 3$, đây là YCBT ban đầu




#407042 ​$(a+b+c)(\frac{1}{a}+\frac{1}...

Đã gửi bởi Atu on 22-03-2013 - 18:17 trong Bất đẳng thức và cực trị

Chứng minh rằng với mọi số thực dương a,b,c ta luôn có
$(a+b+c)(\frac{1}{a}+\frac{1}{b}+\frac{1}{c})\geq \frac{4(a+b+c)^{2}}{ab+bc+ca}-3$

Không hiểu sao dạo này toàn làm tương đương  =='
Biến đổi tương đương, đưa bài toán về:
$\sum_{sym}\frac{a}{b}+6\geq \frac{4\sum a^{2}+8\sum ab}{\sum ab}$
$\Leftrightarrow(\sum ab)(\sum_{sym}\frac{a}{b})\geq 4\sum a^{2}+2\sum ab$
Nhân vế trái ra hết ta được: 
$(\sum ab)(\sum_{sym}\frac{a}{b})=2\sum a^{2}+2\sum ab+\sum \frac{a^{2}b}{c}+\sum \frac{ab^{2}}{c}$
Như vậy chỉ cần cm:
$\sum \frac{a^{2}b}{c}+\sum \frac{ab^{2}}{c}\geq 2\sum a^{2}$
Áp dụng $AM-GM$ ta có:
$\sum \frac{a^{2}b}{c}+\sum \frac{a^{2}c}{b}\geq 2\sum a^{2}$




#406753 $\frac{a^2}{a-b}+\frac{b^2}...

Đã gửi bởi Atu on 21-03-2013 - 18:29 trong Bất đẳng thức và cực trị

Bài 2: Cho $a,b,c  \mathbb{R}$ thỏa mãn $ a>b>c$ CMR

 

$\frac{a^2}{a-b}+\frac{b^2}{b-c}> a+c+2b$

Cái này hình như chỉ là biến đổi tương đương?

$\frac{a^{2}}{a-b}-a-b=\frac{b^{2}}{a-b}$ (1)

$\frac{b^{2}}{b-c}-c-b=\frac{c^{2}}{b-c}$ (2)

(1) và (2) suy ra $\frac{b^{2}}{a-b}+\frac{c^{2}}{b-c}> 0$ (đúng)




#406454 CMR $4(a^{9}+b^{9}) \geq (a+b)(a^{3}+...

Đã gửi bởi Atu on 20-03-2013 - 16:28 trong Bất đẳng thức và cực trị

Cho $a,b,c$ là các số thực thỏa mãn: $a+b \geq 0$

Chứng minh rằng: $4(a^{9}+b^{9}) \geq (a+b)(a^{3}+b^{3})(a^{5}+b^{5})$

Trước hết, ta có bđt phụ sau:

$a^{6}+b^{6}\geq ab(a^{4}+b^{4})\Leftrightarrow (a-b)(a^{5}-b^{5})\geq 0$ (đúng do xét 2 TH $a\geq b$ và $a< b$) (1)

Ta có:

$4\sum a^{9}=4(\sum a^{3})(\sum a^{6}-a^{3}b^{3})$

Do $a+b> 0$ nên $a^{3}+b^{3}\geq 0$ nên ta có thể tương đương bài toán thành:

$4(\sum a^{6}-(ab)^{3})\geq (\sum a)(\sum a^{5})\Leftrightarrow 3\sum a^{6}\geq ab(\sum a^{4})+3(ab)^{3}$ (đúng do (1) và bđt $AM-GM$)




#406354 $\sum a(a-b)(a-c) \ge \dfrac{(a+b+c)^4}{27...

Đã gửi bởi Atu on 19-03-2013 - 21:46 trong Bất đẳng thức và cực trị

Bài toán: Cho $a,b,c >0$ thỏa mãn $abc=1$,chứng minh rằng:

$a(a-b)(a-c)+b(b-a)(b-c)+c(c-a)(c-b) \ge \dfrac{(a+b+c)^4}{27}-2(a^7+b^7+c^7)+3$

Trước hết, áp dụng bđt $Schur$ cho VT ta có:

$VT\geq 0$

Như vậy, cần chứng minh $VP\leq 0$ hay $2\sum a^{7}\geq \frac{(\sum a)^{4}}{27}+3$

Ta có:

$3\sum a^{2}\geq (\sum a)^{2}$ và $\sum a\geq 3$ nên $\sum a^{2}\geq \sum a$

Vậy:

$\sum a^{7}+5\sum a\geq 6\sum a^{2}\geq 6\sum a\Rightarrow \sum a^{7}\geq \sum a$

Bây giờ ta có:

$2\sum a^{7}\geq \sum a^{7}+\sum a\geq 2\sum a^{4}\geq \sum a^{4}+3\geq \frac{9(\sum a^{2})^{2}}{27}+3\geq \frac{(\sum a)^{4}}{27}+3$

 


Hình như $VT \ge 0;VP \ge 0$ thì không khẳng định được $VT \ge VP$

 $VP\leq 0\Leftrightarrow 2\sum a^{7}\geq \frac{(\sum a)^{4}}{27}+3$ SR,anh viết nhầm




#405705 Chứng minh rằng:$2(a^3+b^3+c^3) \leq 3+a^2b+b^2c+c^2a$

Đã gửi bởi Atu on 17-03-2013 - 10:23 trong Bất đẳng thức và cực trị

Cho các số thực a,b,c$\in$[0;1]
Chứng minh rằng:$2(a^3+b^3+c^3) \leq 3+a^2b+b^2c+c^2a$

Do a,b,c$\in$[0;1] nên ta có:
$(a^{2}-1)(b-1)\geq 0\Leftrightarrow a^{2}b+1\geq a^{2}+b$
Tương tự rồi cộng lại ta được:
$\sum a^{2}b+3\geq \sum a^{2}+\sum a$ (1)
Mà a,b,c$\in$[0;1] nên:
$\sum a\geq \sum a^{3};\sum a^{2}\geq \sum a^{3}$ (2)
Từ (1) và (2) suy ra đpcm



#405271 sẽ bổ sung nội dung sau

Đã gửi bởi Atu on 15-03-2013 - 16:05 trong Bất đẳng thức và cực trị

http://diendantoanho...nn-của-mx2y2z2/



#405003 $4(xy+yz+xz)\leqslant \sqrt{(x+y)(y+z)(x+z)}(\s...

Đã gửi bởi Atu on 14-03-2013 - 18:32 trong Bất đẳng thức và cực trị

Đề thi chọn đội tuyển lớp 10 trường THPT chuyên LHP-TPHCM đợt 1 năm nay đây mà :)

Cho $x,y,z>0$. Chứng minh rằng:
$4(xy+yz+xz)\leqslant \sqrt{(x+y)(y+z)(x+z)}(\sqrt{x+y}+\sqrt{y+z}+\sqrt{x+z})$

Spoiler

Cách làm của mình ( có lẽ sẽ hơi dài):
Chuẩn hoá $\sum xy=3$, cụ thể như sau:
Đặt $\sum xy=3t^{2}$ và $a=\frac{x}{t};b=\frac{y}{t};c=\frac{z}{t}$ ; ta sẽ đưa về bài toán:
Cho $\sum ab=3$; CMR: $\sum (a+b)\sqrt{(b+c)(c+a)}\geq 4\sum ab$
Ta có:
$\sum (a+b)\sqrt{(b+c)(c+a)}=\sum (a+b)\sqrt{3+c^{2}}$
$=\sum (a+b)\frac{\sqrt{(3+c^{2})(3+1)}}{2}\geq \sum \frac{(a+b)(c+3)}{2}=\sum ab+3\sum a\geq 4\sum ab$ (do $(\sum a)^{2}\geq 3\sum ab=(\sum ab)^{2}\Rightarrow \sum a\geq \sum ab$)



#404996 Tìm GTNN: $P=\frac{x}{xy+1}+\frac{y...

Đã gửi bởi Atu on 14-03-2013 - 18:09 trong Bất đẳng thức và cực trị

Có ở đây nè
http://diendantoanho...eqslant-frac32/



#404889 $\sum \frac{(s-b)(s-c)}{s-a}=s$

Đã gửi bởi Atu on 13-03-2013 - 22:34 trong Bất đẳng thức và cực trị

Cho $a,b,c$ là độ dài ba cạnh của tam giác $ABC $, $s$ là nửa chu vi tam giác ABC . Định dạng tam giác $ABC$ biết rằng : $\sum \dfrac{(s-b)(s-c)}{s-a}=s$

Áp dụng bđt quen thuộc:
$\sum x^{2}y^{2}\geq xyz(x+y+z)\Rightarrow \sum \frac{xy}{z}\geq \sum x$
Bây giờ, đặt $x=s-a;y=s-b;z=s-c$, rồi áp dụng bđt trên, ta có:
$\sum \frac{(s-b)(s-c)}{s-a}\geq s$
Dấu $"="$ xảy ra $\Leftrightarrow$ a=b=c
Vậy tam giác $ABC$ đều



#404840 Tìm GTNN A=$\dfrac{1}{x^2+y^2}+\dfrac...

Đã gửi bởi Atu on 13-03-2013 - 21:18 trong Bất đẳng thức và cực trị

Cho hai số dương x,y thỏa $x+y\leqslant 1$ . Tìm GTNN A=$\dfrac{1}{x^2+y^2}+\dfrac{2}{xy}+4xy$

Ta có:
$\frac{1}{x^{2}+y^{2}}+\frac{1}{2xy}\geq \frac{4}{(x+y)^{2}}\geq 4$
$4xy+\frac{1}{4xy}\geq 2$
$\frac{5}{4xy}\geq \frac{5}{(x+y)^{2}}\geq 5$
Cộng vế theo vế ta có :
$A\geq 11$



#404474 $\frac{a}{\sqrt{b}} + \frac...

Đã gửi bởi Atu on 12-03-2013 - 18:34 trong Bất đẳng thức và cực trị

1.Cho 3 số thực dương a , b , c thoả a + b + c = 3
Chứng minh $\frac{a}{\sqrt{b}} + \frac{b}{\sqrt{c}} + \frac{c}{\sqrt{a}} \geq 3$



$\sum \frac{a}{\sqrt{b}}+\sum \frac{a}{\sqrt{b}}+\sum ab\geq 3(a+b+c)=9$(1)
Lại có:
$9=(\sum a)^{2}\geq 3\sum ab\Rightarrow \sum ab\leq 3$ (2)
(1) và (2) suy ra đpcm



#403803 $\frac{1}{ab}+\frac{1}{bc}+\frac{1}{ca} \geq 3 +...

Đã gửi bởi Atu on 10-03-2013 - 20:32 trong Bất đẳng thức và cực trị

Cho ab+bc+ca=1
CMR
$\frac{1}{ab}+\frac{1}{bc}+\frac{1}{ca} \geq 3 + \sqrt{\frac{1}{a^{2} }+1} + \sqrt{\frac{1}{b^{2}}+1} + \sqrt{\frac{1}{c^{2}}+1}$


$BĐT$ đã cho tương đương:
$\sum \frac{1}{ab}-3\geq \sum \sqrt{\frac{ab+bc+ca+a^{2}}{a^{2}}}$
Xét $VP$:
$\sum \sqrt{\frac{ab+bc+ca+a^{2}}{a^{2}}}\leq \sum \frac{a+b}{2a}+\sum \frac{a+c}{2c}=\sum \frac{ab+b^{2}}{2ab}+\sum \frac{a^{2}+ab}{2ab}$
$=\sum \frac{(a+b)^{2}}{2ab}$ (1)
Xét $VT$:
$\sum \frac{1}{ab}-3=\sum \frac{1-ab}{ab}=\sum \frac{c(a+b)}{ab}$ (2)

(1) và (2) $\Rightarrow \sum (a+b)(\frac{2c-a-b}{ab})\geq 0$
Đến đây bí quá nên mình làm hơi dài:
Do vai trò a;b;c như nhau nên không mất tính tổng quát giả sử $a\geq b\geq c$
Ta cần CM:
$(a+b)(\frac{2c-a-b}{ab})+(b+c)(\frac{2a-b-c}{bc})+(c+a)(\frac{2b-c-a}{ca})\geq 0$
$\Leftrightarrow (a+b)(\frac{2c-a-b}{ab})+(b+c)(\frac{2a-b-c}{bc})-(a+c)(\frac{2a-b-c+2c-a-b}{ac})\geq 0$
$\Leftrightarrow (2a-b-c)(\frac{b+c}{bc}-\frac{a+c}{ac})+(a+b-2c)(\frac{a+c}{ac}-\frac{a+b}{ab})\geq 0$
$\Leftrightarrow (2a-b-c)(\frac{c(a-b)}{ab})+(a+b-2c)(\frac{a(b-c)}{bc})\geq 0$ (đúng)



#403498 $\frac{AO}{OM}+\frac{BO}{ON...

Đã gửi bởi Atu on 09-03-2013 - 23:11 trong Bất đẳng thức và cực trị

Cho tam giác ABC, O là điểm bất kì trong tam giác, AO, BO, CO kéo dài cắt cạnh đối diện tại M,N,P. Cm:
$\frac{AO}{OM}+\frac{BO}{ON}+\frac{CO}{OP}\geq 6$


Cộng 3 vào mỗi vế đề bài trở thành:
$\frac{AM}{OM}+\frac{BN}{ON}+\frac{CP}{OP}\geq 9$
Đặt $S_{OBC}=a;S_{OAB}=b;S_{OAC}=c$
Chứng minh được:
$\frac{AM}{OM}=\frac{a+b+c}{a};\frac{BN}{ON}=\frac{a+b+c}{c};\frac{CP}{OP}=\frac{a+b+c}{b}$
Đưa về bđt quen thuộc:
$(\sum a)(\sum \frac{1}{a})\geq 9$



#403322 $P=\frac{4x^2+y^2+1}{2x-y}$

Đã gửi bởi Atu on 09-03-2013 - 19:48 trong Bất đẳng thức và cực trị

Cho $x,y$ là các số thực thỏa mãn $xy=2,2x>y$
Tìm Min của $P=\frac{4x^2+y^2+1}{2x-y}$

$2x>y\Rightarrow 2x-y> 0$
Ta có:
$P=\frac{4x^{2}+y^{2}+1}{2x-y}=\frac{4x^{2}-4xy+y^{2}+9}{2x-y}$
$=2x-y+\frac{9}{2x-y}=3\left ( \frac{2x-y}{3}+\frac{3}{2x-y} \right )\geq 6$
Dấu "=" xảy ra $\Leftrightarrow \left\{\begin{matrix} x=2\\y=1 \end{matrix}\right.hay \left\{\begin{matrix} x=-\frac{1}{2}\\ y=-4 \end{matrix}\right.$



#402849 Tìm MIN của $\sum \frac{a^{3}}{b^...

Đã gửi bởi Atu on 07-03-2013 - 21:12 trong Bất đẳng thức và cực trị

$\oplus$ Ta có: $\dfrac{a^3}{b^4+c^4} + \dfrac{b^4+c^4}{4} + \dfrac{1}{2} \ge 3\sqrt[3]{{\frac{{{a^3}.({b^4} + {c^4}).1}}{{({b^4} + {c^4}).4.2}}}} = \frac{{3a}}{2}$
Thiết lập các Bđt tương tự, ta được:
$\sum \dfrac{a^3}{b^4+c^4} + \dfrac{a^4+b^4+c^4}{2} + \dfrac{3}{2} \ge \dfrac{9}{2}$
$\Longleftrightarrow$ $\sum \dfrac{a^3}{b^4+c^4} + \dfrac{a^4+b^4+c^4}{2} \ge \dfrac{9}{2}-\dfrac{3}{2}=3$$(1)$
$\oplus$ Ta có: $\frac{{{a^4} + {b^4} + {c^4}}}{2} \ge \frac{{\frac{{{{({a^2} + {b^2} + {c^2})}^2}}}{3}}}{2} \ge \frac{{\frac{{\frac{{{{(a + b + c)}^2}}}{3}}}{3}}}{2} = \frac{{\frac{{\frac{9}{3}}}{3}}}{2} = \frac{1}{2}$ $(2)$
$\oplus$ Thay $(2)$ vào $(1)$, ta được ĐPCM

Thế vào rồi sao nữa? Cách làm này hình như có vấn đề ở chỗ màu đỏ rồi!



#402306 $a^{2}(b+c-a)+b^{2}(a+c-b)+c^{2}(a+b-c)...

Đã gửi bởi Atu on 05-03-2013 - 20:39 trong Bất đẳng thức và cực trị

Bài1:

Cho a,b,c>0 CMR:

$a^{2}(b+c-a)+b^{2}(a+c-b)+c^{2}(a+b-c)\le 3abc$



Cái này hình như là Shur bậc 3 thì phải:
Giả sử $a\geq b\geq c$
$BĐT$$\Leftrightarrow \sum a^{3}+3abc\geq \sum ab(a+b)\Leftrightarrow a(a-b)(a-c)+b(b-c)(b-a)+c(c-a)(c-b)\geq 0\Leftrightarrow (a-b)\left [ a(a-c)-b(b-c) \right ]-c(c-a)(c-b)\geq 0$
(đúng do a(a-b)-b(b-c)$\geq 0$)

còn cách này:(bđt 22)
http://diendantoanho...hụ/page__st__20



#401833 Cho a;b;c>0 .Tìm min A=$a+b^{2}+c^{3}+\fra...

Đã gửi bởi Atu on 03-03-2013 - 21:26 trong Bất đẳng thức và cực trị

Cho a;b;c>0 .Tìm min A=$a+b^{2}+c^{3}+\frac{1}{abc}$

$A=a+b^{2}+c^{3}+\frac{1}{abc}=6\cdot \frac{a}{6}+3\cdot \frac{b^{2}}{3}+2\cdot \frac{c^{3}}{2}+6\cdot \frac{1}{abc}$
Áp dụng $AM-GM$ 17 số:
$A\geq 17\sqrt[17]{\frac{(abc)^{6}}{6^{6}\cdot 3^{3}\cdot 2^{2}\cdot (abc)^{6}}}=17\sqrt[17]{\frac{1}{6^{6}\cdot 3^{3}\cdot 2^{2}}}$